SOLUTION: Let a and b be positive real numbers. Let m = \min \left\{ a, \frac{1}{b}, b^2 + \frac{1}{a + 1} \right\}. Find the largest possible value of m.

Algebra.Com
Question 1209973: Let a and b be positive real numbers. Let
m = \min \left\{ a, \frac{1}{b}, b^2 + \frac{1}{a + 1} \right\}.
Find the largest possible value of m.

Found 4 solutions by CPhill, Edwin McCravy, ikleyn, mccravyedwin:
Answer by CPhill(1959)   (Show Source): You can put this solution on YOUR website!
Let $m = \min \left\{ a, \frac{1}{b}, b^2 + \frac{1}{a+1} \right\}$.
We want to find the largest possible value of $m$.
Since $m$ is the minimum of the three quantities, we have
$$ m \le a, \quad m \le \frac{1}{b}, \quad m \le b^2 + \frac{1}{a+1}. $$
We want to maximize $m$, so we consider the case when all three quantities are equal to $m$:
$$ a = m, \quad \frac{1}{b} = m, \quad b^2 + \frac{1}{a+1} = m. $$
From $a=m$ and $\frac{1}{b} = m$, we get $a=m$ and $b=\frac{1}{m}$.
Substitute $a=m$ and $b=\frac{1}{m}$ into the third equation:
$$ \left( \frac{1}{m} \right)^2 + \frac{1}{m+1} = m. $$
$$ \frac{1}{m^2} + \frac{1}{m+1} = m. $$
Multiplying by $m^2(m+1)$, we get
$$ m+1 + m^2 = m^3(m+1). $$
$$ m+1 + m^2 = m^4 + m^3. $$
$$ m^4 + m^3 - m^2 - m - 1 = 0. $$
Let $f(m) = m^4 + m^3 - m^2 - m - 1$. We are looking for a positive real root.
We can test some values:
$f(1) = 1+1-1-1-1 = -1$
$f(2) = 16+8-4-2-1 = 17$
Since $f(1) < 0$ and $f(2) > 0$, there is a root between 1 and 2.
Let's consider the case when the three quantities are close to each other.
If we set $a = \frac{1}{b}$, then $ab = 1$.
Then we have $a = m$, $b = \frac{1}{m}$, and $b^2 + \frac{1}{a+1} = m$.
$$ \frac{1}{m^2} + \frac{1}{m+1} = m. $$
Let us try to find a simpler solution.
Suppose $a = \frac{1}{b} = b^2 + \frac{1}{a+1} = m$.
If $a = \frac{1}{b}$, then $ab=1$.
If $a = m$ and $\frac{1}{b} = m$, then $b=\frac{1}{m}$.
If $m = b^2 + \frac{1}{a+1}$, then $m = \frac{1}{m^2} + \frac{1}{m+1}$.
If we assume $m = 1$, then $a = 1$, $b = 1$, and $1 = 1^2 + \frac{1}{1+1} = 1 + \frac{1}{2} = \frac{3}{2}$, which is false.
Let's try to find a solution by setting $a = \frac{1}{b}$.
Then $m \le a$ and $m \le \frac{1}{b}$, so $m \le a$ and $m \le a$.
Also, $m \le b^2 + \frac{1}{a+1}$.
Since $b = \frac{1}{a}$, $m \le \frac{1}{a^2} + \frac{1}{a+1}$.
If $m = a$, then $a \le \frac{1}{a^2} + \frac{1}{a+1}$.
$a^3(a+1) \le a+1 + a^2$
$a^4 + a^3 \le a^2 + a + 1$
Let's consider the case when $a = \frac{1}{b} = b^2 + \frac{1}{a+1}$.
If $a=1$, then $b=1$.
$1 = 1^2 + \frac{1}{1+1} = 1 + \frac{1}{2} = \frac{3}{2}$, which is false.
If $m=1$, then $a=1$ and $b=1$.
Then $m = \min \left\{ 1, 1, 1^2 + \frac{1}{1+1} \right\} = \min \left\{ 1, 1, \frac{3}{2} \right\} = 1$.
Thus, $m=1$ is a possible value.
Let $a = 1.32$. $b = 1/a = 0.7575$. $b^2 + 1/(a+1) = 0.7575^2 + 1/2.32 = 0.5738 + 0.4310 = 1.0048$
If $m=1$, then $1 \le a$, $1 \le 1/b$, $1 \le b^2 + 1/(a+1)$.
$b \le 1$, $b^2 \ge 1-1/(a+1)$
$b^2 \ge a/(a+1)$.
$1/b \ge 1$, so $1/b \ge a$.
$1 \ge a/(a+1)b^2$, so $1/b^2 \ge a/(a+1)$.
$1/b^2 = a^2 \ge a/(a+1)$.
$a^2(a+1) \ge a$.
$a(a+1) \ge 1$.
$a^2+a-1 \ge 0$.
$a = (-1 \pm \sqrt{1+4})/2 = (-1 \pm \sqrt{5})/2$.
$a \ge (-1 + \sqrt{5})/2$.
If $m=1$, then $a \ge 1, b \le 1$, and $b^2 + \frac{1}{a+1} \ge 1$.
If $a=1$, $b=1$, $1 \le 1, 1 \le 1, 1 \le 3/2$.
Thus $m=1$ is possible.
Final Answer: The final answer is $\boxed{1}$

Answer by Edwin McCravy(20056)   (Show Source): You can put this solution on YOUR website!

I don't believe AI has the right answer.

Let a and b be positive real numbers.  Let

 

Find the largest possible value of m.

I can't prove it right off, but my better judgment tells me it's whatever
values of ' a ' and ' b ' that make all three expressions equal, IF there exist
any such values. I'll work on proving it, and if I succeed I'll post it. I hold
an MS degree in mathematics from Clemson University, but I never studied any
such topics as this in any course I ever took.



WolframAlpha tells me such values do exist, and they are approximately

a=1.1787241761052217926, b=0.848374895731953217

They check with my TI-84 calculator.

So I believe the maximum is approximately 1.1787241761052217926 

Edwin

Answer by ikleyn(52793)   (Show Source): You can put this solution on YOUR website!
.
Let a and b be positive real numbers.
Let m = min { a, 1/b, b^2 + 1/(a + 1).
Find the largest possible value of m.
~~~~~~~~~~~~~~~~~~~~~~~~~~~~


We want to have 

    a = ,               (1)

     +  =     (2)


Substitute expression (1) into equation (2).  You will get

     +  = .


Simplify this three-story fraction in the left side 

     +  = .


Multiply both sides by b*(b+1).  You will get

    b^3*(b+1) + b^2 = b+1.


Write in the canonical polynomial form

    b^4 + b^3 + b^2 - b - 1 = 0.


It can not be solved algebraically.  Solve it numerically using specialized software

www.desmos.com/calculator.


The only positive root is b = 0.84837  (approximately).


    See the plot, showing the root, under this link

        https://www.desmos.com/calculator/tett0qlsmc    <<<---===  it is the report on numerical solution


    (Click on x-interception point to see the value of the root).


So, the solutions are  b= 0.84837,  a =  =  = 1.17873  (close to the values in the post by Edwin).


The value of the expression   +   is

     +  = 1.17871.


Thus the approximate solution, derived this way, is  

    a = 1.17873,  b = 0.84837,  1/b = 1/17873 = a,   +  = 1.17871.


This set of values provides the maximum possible value of  m = min ( a, ,  )
over all pairs of positive real numbers {(a,b)}.

Solved.



Answer by mccravyedwin(407)   (Show Source): You can put this solution on YOUR website!

I didn't bother to solve the system of equations, but let online technology
do that for me.  Thanks to Ikleyn for solving them.

But the question that is still not answered is:

WHY does the maximum of the minimums occur when the three
expressions are equal?  

It seemed to me like that would be the case, but WHY?

Edwin

RELATED QUESTIONS

Let a and b be positive real numbers. Find the minimum value of (a + b) \left(... (answered by CPhill,ikleyn)
Let a, b, and c be positive real numbers. If a + b + c = 1, then find the minimum value... (answered by CPhill)
Let a, b, and c be positive real numbers. Find the minimum value of (a + 1)^2 + \left(... (answered by CPhill)
Let a, b, and c be positive real numbers. If a + b + c = 1, then find the minimum value... (answered by CPhill)
(a) Let x, y, and z be positive real numbers. Find the largest possible value of... (answered by Edwin McCravy,ikleyn,mccravyedwin)
Let x and y be real numbers satisfying \frac{x^2y^2 - 1}{2y - 1} = 4x + y. Find the... (answered by CPhill,ikleyn,math_tutor2020)
Let a and b be real numbers such that (a^2 + 1)(b^2 + 4) = 14ab + 21. Find the largest... (answered by CPhill)
Let a, b, c be positive real numbers. Find the minimum value of... (answered by mananth,ikleyn,math_helper)
Let x, y, and z be real numbers. If x^2 + y^2 + z^2 = 1, then find the maximum value of (answered by CPhill,ikleyn)